shirando21
Thanks Received: 16
Atticus Finch
Atticus Finch
 
Posts: 280
Joined: July 18th, 2012
 
 
 

Q2 - Several companies will soon

by shirando21 Fri Sep 14, 2012 4:18 pm

This is a weaken question.

I know how A is correct and B D E are wrong.

I am not sure whether C is wrong because 30% is not big enough or talking about people rely on news programs broadcast on tv and radio cannot attack the argument at all.

If C is written like, 90% of people never.......
will that weaken the argument?
 
timmydoeslsat
Thanks Received: 887
Atticus Finch
Atticus Finch
 
Posts: 1136
Joined: June 20th, 2011
 
This post thanked 1 time.
 
trophy
Most Thanked
trophy
First Responder
 

Re: Q2 - Several companies will soon

by timmydoeslsat Fri Sep 14, 2012 10:14 pm

shirando21 Wrote:This is a weaken question.

I know how A is correct and B D E are wrong.

I am not sure whether C is wrong because 30% is not big enough or talking about people rely on news programs broadcast on tv and radio cannot attack the argument at all.

If C is written like, 90% of people never.......
will that weaken the argument?

Even if C stated 90% of people never relied, we are wanting to talk about those that DO use newspapers. We are concerned with the part of the conclusion that states we will see newspaper sales decline, which is not compromised in any way with how many people currently do not use newspapers.

The problem with the argument is that we determine one factor that favors personalized news over newspapers, yet make an overall judgement of the two based on only this one factor. To weaken the argument, we want to point out anything that is good with newspapers and not so with personalized news. I was expecting cost to be brought up, but (A) did the job nonetheless.
User avatar
 
bbirdwell
Thanks Received: 864
Atticus Finch
Atticus Finch
 
Posts: 803
Joined: April 16th, 2009
 
This post thanked 1 time.
 
 

Re: Q2 - Several companies will soon

by bbirdwell Sat Sep 22, 2012 1:38 pm

Right. (C) applies only to people who don't read newspapers in the first place.

Our conclusion is that newspaper sales will decline. Therefore, people that don't read newspapers right now are beyond our concern.

And, as Timmy pointed out, we want an answer that makes it LESS likely that newspapers sales will decline due to the see-what-you-want nature of new services.

(A) says that sometimes people want unexpected news, which hurts the argument's reasoning.
I host free online workshop/Q&A sessions called Zen and the Art of LSAT. You can find upcoming dates here: http://www.manhattanlsat.com/zen-and-the-art.cfm
 
griffin3575
Thanks Received: 3
Vinny Gambini
Vinny Gambini
 
Posts: 14
Joined: June 21st, 2013
 
 
 

Re: Q2 - Several companies will soon

by griffin3575 Sun Dec 15, 2013 2:21 pm

I understand the reasoning behind C being out of scope, but I had a different reason for eliminating C that I thought I would share:

Notice that the conclusion of the argument states that newspaper sales will drastically decline if the new service becomes widely available. Thus, an assumption the author is making is that newspapers serve a large enough size of the population that sales have the potential to be drastically reduced. In other words, the author is assuming that don't serve a very small segment of the population, say 1%, that is incapable of have a drastic cut in sales**

So, I think C plays on this assumption. By stating that 30% of the population never uses the newspaper for information, it implies that 70% of the population have used the newspaper for information at least once. Thus, newspaper sales have the potential to decline drastically from a maximum potential audience of 70% of the population. Therefore, C closes a very small gap in the argument and strengthens it--> an opposite answer!

Is this solid reasoning?

**Note: It is possible that even if newspaper sales served only 1% of the population, sales could still drastically decline if you look at it as percentage decline in sales. For example, even if they only serve 1% of the population, their sales could still decline 90%. But if you look at it from a monetary standpoint and think that serving only 1% of some population only brings in say $2000 in revenues, a decline in sales of $1000 is not very dramatic.**
User avatar
 
tommywallach
Thanks Received: 468
Atticus Finch
Atticus Finch
 
Posts: 1041
Joined: August 11th, 2009
 
 
 

Re: Q2 - Several companies will soon

by tommywallach Fri Dec 20, 2013 2:15 am

Hey Griffin,

Hmm. I don't love that reasoning. Any sales above 2 can be reduced drastically (if you had 3 copies, removing 2 of them would be a drastic reduction). And a paper couldn't really be a paper with sales below 2 copies!

You seem to be misdefining "drastic" here. It wouldn't be a function of how MANY papers, but of a PROPORTION of papers.

Hope that helps!

-t
Tommy Wallach
Manhattan LSAT Instructor
twallach@manhattanprep.com
Image
User avatar
 
lolitatrekkie
Thanks Received: 0
Jackie Chiles
Jackie Chiles
 
Posts: 26
Joined: June 21st, 2016
 
 
 

Re: Q2 - Several companies will soon

by lolitatrekkie Sun Jul 31, 2016 5:24 am

Can anyone explain why answer choice D is wrong?

Is it wrong because even if the monthly cost of subscribing to several channels on a personalized electronic newspaper will equal the cost of a monthly subscription to a newspaper it doesn't challenge the conclusion since we are trying to prove that newspaper sales won't decline?

Thank you!
"Dearly beloved we are gathered here today to get through this thing called life.."~ Prince
 
VendelaG465
Thanks Received: 0
Elle Woods
Elle Woods
 
Posts: 66
Joined: August 22nd, 2017
 
 
 

Re: Q2 - Several companies will soon

by VendelaG465 Tue Jan 09, 2018 4:36 pm

Why couldn't C work ? I had picked it b/c it stated that people never read newspapers to begin with. So I figured that hurt the conclusion that newspaper would decline if they weren't big/existing to begin with ?
User avatar
 
ohthatpatrick
Thanks Received: 3805
Atticus Finch
Atticus Finch
 
Posts: 4661
Joined: April 01st, 2011
 
 
 

Re: Q2 - Several companies will soon

by ohthatpatrick Sun Jan 14, 2018 2:40 am

I can't tell you how crucial it is to differentiate between COMPARATIVE and ABSOLUTE terms on LSAT.

If a conclusion says "Sales will decline", then it is correct if sales go down by even a penny.

If a conclusion says "Sales will be terrible", then sales may go down but still not be terrible.

That's the difference between COMPARATIVE and ABSOLUTE.
The Eiffel Tower is shorter than the skyscrapers in Dubai.
The Eiffel Tower is not short.

When this argument concludes "sales will decline drastically", I interpret that in terms of rate of sales. If newspapers only sell 20% as many papers as before, we would call that a drastic decline in sales.

"Decline in sales" is probably always interpreted that way.

"Suffer a great loss" is more likely to be interpreted in an absolute term.
I might lose 50% of my savings, but not suffer a great loss if I only had $2 in the bank."

So your interpretation of (C) that the newspaper was never big to begin with is ineligible for this conversation.

The conversation is about whether sales would be drastically less than what they currently are, not whether they would be big or small in absolute terms.